Change of Variables: Integral of x^2+y^2 in Region B

In summary, Rocket is working on a problem involving the Implicit Function Theorem and the Inverse Function Theorem. He is unsure about how to do the first step, which is to find the Jacobian. He has looked at the theorem, but is unsure about how to find the inverse function. He has plotted the results of the three double integrals he mentioned, and found that they all add up to 3.
  • #1
rocket
10
0
Hi,
I'm not sure how to do this question. Any help would be great.

Let B be the region in the first quadrant of [tex] R^2 [/tex] bounded by [tex] xy=1, xy=3, x^2-y^2=1, x^2-y^2=4. [/tex] Find [tex] \int_B(x^2+y^2) [/tex] using the substitution [tex]u=x^2-y^2, v=xy. [/tex]. Use the Inverse Function theorem rather than solving for x and y explicitly.
 
Physics news on Phys.org
  • #2
what have you done? did you calculate the jacobian?
 
  • #3
rocket said:
Hi,
I'm not sure how to do this question. Any help would be great.

Let B be the region in the first quadrant of [tex] R^2 [/tex] bounded by [tex] xy=1, xy=3, x^2-y^2=1, x^2-y^2=4. [/tex] Find [tex] \int_B(x^2+y^2) [/tex] using the substitution [tex]u=x^2-y^2, v=xy. [/tex]. Use the Inverse Function theorem rather than solving for x and y explicitly.

Hello Rocket. Welcome to PF. You know, I've been working with the Implicit Function Theorem lately too and feel this problem nicely accentuates my work. This is what I think and I hope those in here better than me will correct anything I say that reflects my recent understanding of the theorems. :smile:

The problem here is two fold: Calculating the correct Jacobian and then figuring out what are the new limits of integration in terms of u and v. Have you looked at the theorem and especially the case when the transformation is from [itex]\mathbb{R}^2\rightarrow\mathbb{R}^2[/itex]? That's the case above since we have (x,y) to (u,v). In this case, the Implicit Function Theorem reduces to the Inverse Function Theorem which under suitable restrictions, (Jacobian not zero), guarantees the existence of an inverse function. If we have an inverse function, then we have one-to-one and thus:

In the case above we have:

[tex]f:(x,y)\rightarrow (x^2-y^2,xy)[/tex]

In order to solve the indicated integral by change of variable, we require the inverse functions:

[tex]x=h(u,v)[/tex]

[tex]y=k(u,v)[/tex]

and the Jacobian:

[tex]\frac{\partial(x,y)}{\partial(u,v)}[/tex]

However, if the transformations above satisfy the requirements of the Implicit Function Theorem, i.e., are one-to-one in the requested domain, we can solve for this Jacobian using the following relationship:

[tex]\frac{\partial(x,y)}{\partial(u,v)}=\frac{1}{\frac{\partial(u,v)}{\partial(x,y)}}[/tex]

Now, how do we figure out the new bounds on the integral? How about substituting the values of u and v into the four equations?
 
  • #4
Rocket, where you at with this anyway? Can you break it up into three regular double integrals, work it out and then compare the results with the results obtained via change of variables? You like doin' extra work? Big negatory? Anyway, I have a slight problem with the Inverse Function Theorem and the substitution:

f(h(u,v),k(u,v)))

that's needed to do the integration. I interpret this, from the IFT, to be equivalent to f(x,y) and work accordingly but I'm a bit unsure. Seems to work (agrees with the three integrals above), but still I've never done one of these from the perspective of the Inverse Function Theorem. Hopefully we'll get some more replys and we can both learn from them. :smile:

Edit: Oh yea, here's the plot and the intervals for the 3 doubles if you're interested. You can calculate those 4 points right?
 

Attachments

  • covintegral.JPG
    covintegral.JPG
    7.4 KB · Views: 347
Last edited:
  • #5
saltydog said:
Rocket, where you at with this anyway? Can you break it up into three regular double integrals, work it out and then compare the results with the results obtained via change of variables? You like doin' extra work? Big negatory? Anyway, I have a slight problem with the Inverse Function Theorem and the substitution:

f(h(u,v),k(u,v)))

that's needed to do the integration. I interpret this, from the IFT, to be equivalent to f(x,y) and work accordingly but I'm a bit unsure. Seems to work (agrees with the three integrals above), but still I've never done one of these from the perspective of the Inverse Function Theorem. Hopefully we'll get some more replys and we can both learn from them. :smile:

Edit: Oh yea, here's the plot and the intervals for the 3 doubles if you're interested. You can calculate those 4 points right?

What are you unsure about? The 3 double integrals you talk about add up to 3 by my calculation (using the services of http://www.quickmath.com ). Using the Inverse Function Theorem as you posted it leads to a constant function integrated over the variables u and v, which were chosen to make all the integration limits constant; the integral becomes a constant times the area of a rectangle and yields consistent results.
 
  • #6
Hello Dan, This is what:

Letting:

[tex]T:\mathbb{R}^2\rightarrow\mathbb{R}^2\quad\text{where:}[/tex]

[tex]T(x,y)=(x^2-y^2,xy)=(u(x,y),v(x,y))[/tex]

We find:

[tex]\int_B F(x,y)dxdy=\int_A F[h(u,v),g(u,v)]\frac{\partial(x,y)}{\partial(u,v)}dudv[/tex]

with:

[tex]x=h(u,v)[/tex]

[tex]y=g(u,v)[/tex]

However, we do not determine h(u,v) and g(u,v) directly and so I'm left with the integral:

[tex]\int_A (x^2+y^2) \frac{1}{2(x^2+y^2)} dudv[/tex]


It's the substituting of F(x,y) for F[h(u,v),g(u,v)] and the awkwardness of having x,y and u and v in the same integral. What am I doing wrong?
 
  • #7
saltydog said:
[tex]\int_A (x^2+y^2) \frac{1}{2(x^2+y^2)} dudv[/tex]


It's the substituting of F(x,y) for F[h(u,v),g(u,v)] and the awkwardness of having x,y and u and v in the same integral. What am I doing wrong?
Ummm. Don't the x^2+y^2 cancel, leaving no x and y dependence in the integrand?
 
  • #8
krab said:
Ummm. Don't the x^2+y^2 cancel, leaving no x and y dependence in the integrand?


Yes, yes, I know that Krab. And I remember you helping me with Hurkyl's tricky integral some time ago too. Thanks. :smile: With regards to the above, just the fact that they're there and I have to use expressions in terms of x and y is what I find a bit awkward. I realize the best way out of this is to work 10 or 12 more. Perhaps I should look at a few more anyway. :smile:
 
  • #9
Ok, I have a good question: Suppose they wouldn't cancel. Suppose we had this:

[tex]\int_A (x^2+y^3) \frac{1}{2(x^2+y^2)} dudv[/tex]

Then what? Would we then be reduced to finding explicit expressions for x and y in terms of u and v I suppose?

Suppose we couldn't find explicit expressions? Could we still rely on the Inverse Function Theorem to solve the problem?

Rocket, where you at anyway? Hope I didn't run you off by asking you to do those extra 3 integrals. Just a suggestion you know. :smile:
 
Last edited:
  • #10
saltydog said:
Ok, I have a good question: Suppose they wouldn't cancel. Suppose we had this:

[tex]\int_A (x^2+y^3) \frac{1}{2(x^2+y^2)} dudv[/tex]

Then what? Would we then be reduced to finding explicit expressions for x and y in terms of u and v I suppose?

Suppose we couldn't find explicit expressions? Could we still rely on the Inverse Function Theorem to solve the problem?

If the x,y dependence did not divide out, you would have to find x(u,v) and y(u,v) in order to perform the integral over u,v. The theorem would still apply as long as the one-to-one condition was satisfied, but it would not be as useful as in the original problem.
 

1. What is the purpose of a change of variables in this integral?

The purpose of a change of variables is to simplify the integral by transforming it into a new coordinate system. This can make the integral easier to evaluate or can help to express the integral in terms of a different variable.

2. How do you determine the new variable to use in a change of variables?

The new variable is typically chosen based on the shape and boundaries of the region B in which the integral is being evaluated. The goal is to choose a variable that transforms the region into a simpler shape, such as a rectangle or circle.

3. Can any type of function be used as the new variable in a change of variables?

No, the new variable must be a one-to-one transformation. This means that every point in the original region B must correspond to exactly one point in the new coordinate system, and vice versa. This ensures that the integral is still evaluated over the same region.

4. How does a change of variables affect the limits of integration in the integral?

The new limits of integration are determined by substituting the new variable into the original limits and solving for the new values. The new limits may be expressed in terms of the original variable or may use the new variable.

5. Can a change of variables be used to solve any integral?

No, a change of variables is only useful in certain cases where it simplifies the integral. It is important to carefully consider the shape and boundaries of the region B before deciding to use a change of variables. In some cases, it may not be beneficial or may not even be possible to use a change of variables.

Similar threads

  • Introductory Physics Homework Help
Replies
25
Views
1K
  • Introductory Physics Homework Help
2
Replies
40
Views
860
Replies
2
Views
260
  • Introductory Physics Homework Help
Replies
1
Views
866
  • Introductory Physics Homework Help
Replies
4
Views
2K
  • Calculus and Beyond Homework Help
Replies
6
Views
539
  • Introductory Physics Homework Help
Replies
3
Views
714
  • Calculus and Beyond Homework Help
Replies
3
Views
911
  • Introductory Physics Homework Help
Replies
7
Views
1K
  • Calculus
Replies
29
Views
685
Back
Top